LSAT and Law School Admissions Forum

Get expert LSAT preparation and law school admissions advice from PowerScore Test Preparation.

 mandrewg
  • Posts: 11
  • Joined: Dec 29, 2017
|
#43184
#3 is considered a StrengthenX, however, from the wording I initially sought it out as a Justify ConclusionX. How would I rule this out as a Justify Conclusion versus the StrengthenX?
User avatar
 Jonathan Evans
PowerScore Staff
  • PowerScore Staff
  • Posts: 726
  • Joined: Jun 09, 2016
|
#43185
Hi, Mandrewg,

Good question! Clearly you've correctly identified "properly drawn" as indicator words for a Justify the Conclusion question. This is correct. However, there is a significant distinction between Justify and Strengthen questions that has to do with degrees of logical certainty.

Strengthen Questions ask you to find an answer that most supports the conclusion of the argument in the stimulus.

(Note the word "most" for now)

Justify Questions ask you to find an answer that if true would be sufficient to justify the conclusion of the argument in the stimulus.

Note the distinction between these two tasks. The difference is not just about word choice. The difference concerns whether or not the correct answer suffices to make the argument necessarily valid.

The correct answer to a Strengthen question will be a statement that is most helpful to the argument among the options given. In other words, it will be better than the other choices. However, the correct answer need not be strong enough to make the argument valid.

It's not good enough for the correct answer to a Justify question only to make the argument stronger. The correct answer to a Justify question must be so helpful that it makes the argument airtight, incontrovertible, valid.

Let's now bring this discussion into StrengthenX and JustifyX.

Let's begin with a Justify question stem and then convert it into a JustifyX question:
  • Justify Question: Which of the following, if true, establishes that the conclusion is properly drawn?

    JustifyX Question: All of the following, if true, establish that the conclusion is properly drawn EXCEPT:
Note that in the JustifyX question there is no variation in degree. It's all or nothing. Either the statement in an answer is sufficient to guarantee the validity of the conclusion or it is not. The one answer that is not sufficient is the correct answer to a JustifyX question.

Now let's look at a Strengthen question stem and then convert it into a StrengthenX question:
  • Strengthen Question: Which of the following, if true, is most helpful in establishing that the conclusion is properly drawn?

    StrengthenX Question: Which of the following, if true, is LEAST helpful in establishing that the conclusion is properly drawn?
We have kept the language very similar but we've changed the question task. Note the Strengthen and StrengthenX questions deal with "how helpful" answer choices are. For the Strengthen question, which is the most helpful? For the StrengthenX question, which is the least helpful?

I hope this was helpful! :-)
 mandrewg
  • Posts: 11
  • Joined: Dec 29, 2017
|
#43186
Yes! This explanation was amazing. In order to distinguish between the two stems (for myself) I needed to apply the varying degrees in which a question stem has an answer choice that would be "most helpful" (strengthen) versus the answer choice that can ultimately serve as the be-all-end-all validity to a conclusion. Wow, it's like a small distinction, but it was absolutely helpful in solidifying my understanding of the two concepts. Thank you, Jonathan!

Get the most out of your LSAT Prep Plus subscription.

Analyze and track your performance with our Testing and Analytics Package.